6
$\begingroup$

What does it tell you about two functions if their $L^p$ norms are the same for all $p\in[1,\infty]$? Certainly they could be related by composition with a diffeomorphism with Jacobian of norm 1, or even one could be a "pulled apart version of the other one" in the sense of $x^2\chi_{[0,2]}$ vs $x^2 \chi_{[0,1]} + (x-1)^2 \chi_{[2,3]}$. To try to ignore the second type of issue, I'll restrict to smooth functions, and ask the following precise question:

Given $f,g\in C^\infty(\mathbb{R})$ such that $\Vert f \Vert_{L^p(\mathbb{R})} = \Vert g \Vert_{L^p(\mathbb{R})} <\infty$ for all $p\in [1,\infty]$ is it necessarily true that $f(x) = g(\pm x + C)$ for some constant $C$ and for a choice of either $+$ or $-$ for all $x \in \mathbb{R}$?

As Qiaochu Yuan shows in his answer, smoothness doesn't solve the issue of "pulling apart" at all. Thus, I am interested in the following:

What is the "smallest" (in whatever sense) but still nonempty subset of $S \subset [1,\infty]$ such that there is $f,g\in C^\infty(\mathbb{R})$ such that $S$ is the set of $p$ such that $\Vert f \Vert_{L^p} \neq \Vert g \Vert_{L^p}$?

$\endgroup$
1
  • $\begingroup$ It's not clear to me whether the complement of S can even contain an interval. Does anyone have such an example? $\endgroup$ Nov 11, 2010 at 0:14

2 Answers 2

12
$\begingroup$

The complement of $S$ in $(1,\infty)$ can't contain an interval or even a sequence converging to a point in $(1,\infty)$. Let $f$ and $g$ be two real functions on ${\mathbb{R}}$ all whose moments exist. Assume $\int|f|^p=\int|g|^p$ for all $p\in S^c$. put $h(z)=\int|f|^z-\int|g|^z$. where $z\in \mathbb{C}$. $h$ is an analytic functions in $\{Real z>1\}$ which we assume is not constantly zero (since $S$ is not empty) so can't vanish on a converging sequence.

$\endgroup$
0
2
$\begingroup$

No. You can take $f, g$ to be sums of smooth bump functions with disjoint support. As long as $f, g$ each have bumps of the same size and shape you can put them in arbitrary locations (and orientations). Maybe you should require something like $f, g$ non-negative everywhere.

$\endgroup$
2
  • $\begingroup$ Ok, well I see that I've failed to remove the "pulling apart" issue.. $\endgroup$ Nov 10, 2010 at 21:00
  • $\begingroup$ I've edited the question to see if there is any insight on the second question. $\endgroup$ Nov 10, 2010 at 21:03

Your Answer

By clicking “Post Your Answer”, you agree to our terms of service and acknowledge you have read our privacy policy.

Not the answer you're looking for? Browse other questions tagged or ask your own question.